Please answer the question in the picture.

Please Answer The Question In The Picture.

Answers

Answer 1

Answer:

Yes there is an error

Step-by-step explanation:

Answer 2

Answer:

Yes there is a mistake

Step-by-step explanation:


Related Questions

Una escalera está apoyada sobre la pared que mide 3.5m. El pie de la escalera hasta la pared mide 2.5m ¿Cuánto mide la escalera?

Answers

Responder:

Explicación paso a paso:

La configuración formará un triángulo de ángulos rectos.

Deje que el pie de la escalera a la pared sean los lados adyacentes = 2,5 m

La altura de la pared será el lado opuesto = 3,5 m

La longitud de la escalera será la hipotenusa.

Usando el teorema de Pitágoras:

hip² = opp² + adj²

hip² = 3,5² + 2,5²

hip² = 12,25 + 6,25

hip² = 6

hip = √6

hip = 2,45 m

Por tanto, la longitud de la escalera es 2,45 m.

A kayak is originally priced at $160. The online retailer gives a discount and the kayak is now priced at $88. Enter the percentage discount for the cost of the kayak.

Answers

Answer:An important category of percentage exercises is markup and markdown problems. For these, you calculate the markup or markdown in absolute terms (you find by how much the quantity changed), and then you calculate the percent change relative to the original value. So they're really just another form of "increase - decrease" exercises.

Step-by-step explanation:

Answer:

45%

Step-by-step explanation:

O= 160

C=88

160-88=72

72       ?

160    100

72 times 100=7200

7200/160=45

Write a question that represents the following equation.
22-7= 15

Answers

Answer:

15 + 7= 22

Step-by-step explanation:

Expand.
Your answer must be a polynomial in standard form.
m(-m^3 + m^2 + 3m)

Answers

Answer:

-m^4+m^3+3m^2

Step-by-step explanation:

Is this equation linear?
y=x^2+3?

Is this equation linear?
y= x/4+7 (The x/4 is a fraction)

Answers

9514 1404 393

Answer:

noyes

Step-by-step explanation:

In the equation ...

  y = x² +3

the largest variable exponent is 2, which is not 1. The equation is not linear.

__

In the equation ...

  y = (1/4)x +7

the largest variable exponent is 1, so the equation is linear.

What does the arrow represent at the end of the graph
Your answer:
Goes on forever
Stops at the arrow
You didn't go over it

Answers

The arrow at the end of a graph tells us that it goes on forever.

Please Please answer this! I will be so thankful and will mark brainliest.

If angle 1 and angle 3 are vertical angles, and m<1= (5x + 14) degrees and m<3= (7x - 4) degrees, what is the value of x, and the measure for angle 1?

Answers

Answer:

x = 9

[tex] m\angle 1= 59\degree [/tex]

Step-by-step explanation:

Since, [tex] \angle 1 \:and \:\angle 3[/tex] are vertical angles.

[tex]\therefore m\angle 1 = m\angle 3\\

\therefore (5x + 14) \degree= (7x - 4) \degree\\

\therefore (5x + 14) = (7x - 4) \\

\therefore 5x - 7x = - 4 - 14\\

\therefore -2 x = - 18\\\\

\therefore x =\frac{-18}{-2}\\\\

\huge \orange {\boxed {\therefore x = 9}} \\\\

m\angle 1 = (5x + 14) \degree\\

m\angle 1 = (5\times 9+ 14) \degree\\

m\angle 1 = (45+ 14) \degree\\

\huge \purple{ \boxed{m\angle 1 =59\degree}}[/tex]

Oscar received a $100 gift card from Better Buy for his birthday. He used the card to purchase 3 DVDs. The balance on the card is now $74.50. If all the DVDs cost the same amount, what was the cost, x, of 1 DVD?

Answers

Answer:

24.83

Step-by-step explanation:

74.50 divided by 3

What is the median of the data set?
42, 78, 42, 53, 60

Answers

Answer:

53

Step-by-step explanation:

first put it in least to greatest

42,42,53,60,78

the middle value is the median

2b-4/3=b-3/6. I don’t know how to solve it :(

Answers

Answer: b=5/6

You want to get b on one side and the fractions on the other. First you should subtract b from both sides giving you 2b-b-4/3=-3/6. Then you add 4/3 to both sides. That gives you 2b-b=-3/6+4/3. Combine like terms. Subtract b from 2b. Find the least common denominator for the fraction side which would be 6. This makes the equation b=-3/6+8/6. Add the fractions and you get b=5/6
b= 5/6
Hope it helps

Slope intercept equation

Answers

Sorry I’m confused! Maybe....

simplify

7 + 2 (4x - 3) = ?

Answers

Calculate 2 times 4x is 8x and 2 times -3 is -6 it should be 7+8x-6 from there you move the six to the seven and it should be 1+8x
the answer will be 1 + 8x

Solve the system by substitution. Check your solution.
y = 0.8 x + 7.2.
20 x + 32 y = 48.
a. (8, -6)
b. (5, 11)
c. (-4, 4)
d. (-10, 2)

Answers

Answer:

c.  (-4, 4)

Step-by-step explanation:

y = 0.8x + 7.2

20x + 32y = 48

20x + 32y = 48

20x + 32(0.8x + 7.2) = 48

20x + 25.6x + 230.4 = 48

45.6x + 230.4 = 48

45.6x = -182.4

x = -4

y = 0.8x + 7.2

y = 0.8(-4) + 7.2

y = -3.2 + 7.2

y = 4

Answer:

C. (-4,4)

Step-by-step explanation:

Because i did it and got it right on (Edge)


what is a negative number that has an absolute value greater than 10​

Answers

Answer:

-11

Step-by-step explanation:

any number less than -10 will have an absolute value higher than 10

Tangela earns the same amount each week by babysitting. When she started this job, she had $200 in her savings account.

The table shows Tangela's savings account balance for her first month of work.

Week 0 1 2 3 4
Balance $200 $350 $500 $650 $800

Let x represent the number of weeks Tangela has been working and let y represent Tangela's account balance.

Which equation, expressed in Slope-Intercept form, represents the amount of money in Tangela's bank account over time?

Answers

Answer:

y = 150x + 200

Step-by-step explanation:

Given the table :

Week (x) __ 0 ____ 1 ____ 2 ____3 _____ 4

Balance(y) :$200_ $350_ $500_ $650_ $800

Equation in slope - intercept form:

y = mx + c

Where

x and y are the independent(x) and dependent variables (y)

C = intercept, value of y, when x = 0

m = gradient change in y per change in x

Hence,

c = $200 ; y value when x = 0

m = change in x / change in y

m = (500 - 350) / ( 2 - 1)

m = $150

Hence, equation becomes :

y = 150x + 200

12,851 rounded to the nearest ten thousand?

Answers

Answer:

13000

Step-by-step explanation:

because 851 is above 500 you add 1 to the 2 then put 000 for the other numbers

843 ÷ 10 To solve 843 ÷ 10, start by showing 843 on the place value chart. thousands hundreds tens ones tenths hundredths thousandths 1 2 3 4 5 6 7 8 9 0

Answers

Answer:

84.3

Step-by-step explanation:

843 ÷ 10

The place value of 843 is

8 = Hundreds

4 = Tens

3 = Units or ones

Hence,

843 ÷ 10

= 843/10

= 84.3

Find the height of a right rectangular prism with surface area 286 m 2 , length 10 m, and width 8 m.

please help qwq

Answers

the answer is 17

286 m 2/10x8

The dimensions of the box below are reduced by half. What is the constant of proportionality between the volume of the new box and the volume of the original box? Alternative text A.1/2 B.1/4 C.1/8 D.1/16

Answers

Answer:

C. 1/8

Step-by-step explanation:

Dimensions of the Original Box:

Length = 40 inches

Width = 20 inches

Height = 8 inches

Volume of original box = L*W*H = 40*20*8 = 6,400 in³

Dimensions of the New Box (½ of the dimensions of the original box):

Length = ½*40 = 20 in

Width = ½*20 = 10 in

Height = ½*8 = 4 in

Volume of original box = L*W*H = 20*10*4 = 800 in³

Constant of proportionality between the volume of the new box and the volume of the original box = [tex] \frac{800}{6,400} = \frac{1}{8} [/tex]

(x+12)23-(2x-3)+16=12+5x-1
hmmmmmmmmm

Answers

Answer:

look at the attached picture

Answer:

X = -16.75

Step-by-step explanation:

Distribute.

(x + 12)23 = 23x + 276

12 + 5x -1 = 11 + 5x

Combine like terms.

23x - 2x - 5x = 16x

11 - (276 + 3+16 ) = -284

16x = -284

x = -17.75

-3(m - 4) <6
Solve inequality

Answers

isolate the variable by dividing each side by factors that dont contain the variable inequality form:

M > 2

You decided to get the Cinemark Movie Pass. It is a flat
fee of $8 and an additional $11.00 per ticket.
Enter the equation of the line in y= mx + b form.

Answers

i think it’s y= 11x + 8 :) sorry it it’s wrong !!

Answer:

y = 11t + 8

Step-by-step explanation:

Since the starting fee is 8 dollars and the additional costs are 11 dollars per ticket, you can write the equation as:

y = 11t + 8

Where "t" represents the amount of tickets bought.

Help! Pls! I'm lost. I would really appreciate it if anyone knows all the answers.

(Math; Scientific Notations)

Answers

Answer:

1) b. 1.562x10

2) a. 4.8x10

4) d. 2.3x10³

5) a. 4.21x10 -

6) 6.6835x10

Step-by-step explanation:

1) (2.2x10)(7.1x10³)= 156200000

2) 9.6x10÷2x10³= 4800000

4) 6.9x10= 2300

5) 5.6x10 -⁷ + 3.65x10 -= 4.21x10 -

6) 6.72x10 - 3.65x10³= 668350

PLZ HELP LAST QUESTION given the piecewise function given below select all statements that are true f(x)=-x+1,x<0 -2,x=0 x^2-1,x>0 All in brackets A.F(4)=7 Bf(-1)=2 c.F(-2)=0 d.F(1)=0

Answers

Answer:

Step-by-step explanation:

Given the function  f(x)=-x+1,x<0 and f(x) = x^2-1,x>0

when x = 4

f(4) =  -4+1

f(4) =  -3

Also f(4) = 4^2 -1

f(4) = 8-1

f(4) = 7

when x = -1

f(-1) =  -(-1)+1

f(-1) =  2

Also f(-1) = (-1)^2 -1

f(-1) = 1-1

f(-1) = 0

when x = -2

f(-2) =  -(-2)+1

f(-2) =  3

Also f(-2) = (-2)^2 -1

f(-2) = 4-1

f(-2) = 3

when x = 1

f(1) =  -1)+1

f(1) =  -2

Also f(1) = (1)^2 -1

f(1) = 1-1

f(1) = 0

From the calculation above, the correct statements are

A. F(4)=7

B f(-1)=2  

d. F(1)=0

-|5a+3(2ab-1)if a=-2and b=-3

Answers

Answer:

-23

Step-by-step explanation:

56 = 7.42-1 Type your answer into the box​

Answers

i dont understand the question

please simplify if possible -5 1/3 × -2 2/5 AND SHOW WORK If you dont know how to show work then dont answer in really stressed and I bbh have 3 tests and 7 assignments due in a few hours ​

Answers

Answer:

-5 1/3 × (-2 ) 2/5

- 16/3 x (-2) 2/5

16/3 x 2 x 2/5

(calculate the product)

64/15

alternate form:

4 4/15

and

4.26

solve for X -6+9x 7x+12


Parallel and Perpendicular lines​

Answers

Answer:

[tex]=63x^2+6[/tex]

Step-by-step explanation:

[tex]-6+9x\cdot \:7x+12[/tex]

[tex]9x\cdot7x=63x^2[/tex]

[tex]=-6+63x^2+12[/tex]

[tex]=63x^2-6+12[/tex]

[tex]=63x^2+6[/tex]

Which of the following statements is true?

6.800 > 6.8
8.2 = 8.002
1.25 > 1.035
7.4 < 7.32

Answers

[tex]1.25 > 1.035[/tex]

100% correct answer.....

Thank you ☺️

Noah and his children went into a grocery store and where they sell apples for
$1 each and mangos for $2 each. Noah has $20 to spend and must buy at
least 9 apples and mangos altogether. If Noah decided to buy 4 apples,
determine the maximum number of mangos that he could buy. If there areno
possible solutions, submit an empty answer.

Answers

Answer:

8 is the maximum he can get.

Step-by-step explanation:

Good luck!

Answer: 8 is the most possible mangos he could buy!



Hope this helps!
Other Questions
Consider the function f(x) = 4x - 7 . What is f(7) ? Choose the word or expression that does not fit 3. - bientt. - demain. - tout l'heure. - Enchant. write two important pieces of info on the role of fats Which statement explains an advantage of a monopoly?It can help small businesses get started.It can more efficiently produce goods.It can ensure that prices are set by the market.It can increase competition in the economy. 2. SPECULATING Why doyou think farming methodsdeveloped in Europeancountries were successful inthe Middle Colonies?HURRY PLZ THANLS Enter the equation in slope intercept form. The line passes through (3,2) and (4,6). The equation of the line is ? Jean, who passed away in 1988, had published a popular book in 1958 through a New York-based publisher. Which words or phrases complete the sentence given below? As per US copyright laws, Jeans copyright ends in BLANK. Until then, if other people use parts of Jeans book in their work, then they need to obtain permission from BLANK.First blank: A.1988 B.2028 C.2058Second blank: A.Jean or her next of kin B. the US Copyright Office C. Jean's publisher 3(2x+5)=2(3x+3) solve for x Compare and contrast Viking and Polynesian exploration and settlement. 11. Name a fictional character who could be described as taciturn. Why would you describethem this way? Find the value of xA - 75C. 75B-15D. 15 Determine the values of m, a, p, q and r. A factory makes 2.3 feet of ropeper minute. How many feet of ropewill the factory make in 7 minutes? quotients to 2 divide by 2 A rectangular swimming pool measures 60 meters in length and 30 meters in width. Joey creates a scale drawing of this pool using the scale 5 cm : 12 m. Find the length, width and perimeter of the pool in Joey's scale drawing. Be sure to include units in your answer. 21 - (3 + 5 + 8) = 21 - 16 5 Miles pls respond ............. please help this question is making me angry it keeps saying it's wrong please I'm begging you Escoge todas las respuestas correctas: Which of the following actions may a bus driver do? aparcar el autobs conducir en la carretera tomar los nidos ir a la estacin Describe how the Mughal culture combined the traditions of several different cultures. PLEASE HELP ILL GIVE Brilliance or whatever its called